PT97.S1.Q24 - Drive-in movie theater

isdmyungisdmyung Member
edited December 2020 in Logical Reasoning 121 karma

I do not even know where to begin with the ACs.
I understand the stimulus as a theater recently closed because the owner claimed that it could not regularly sustain an audience. Then a week prior to the closure, massive profit resulted.
None of the ACs to me make any close parallel to the flaw that I am clearly not able to identify in this question.
I was thinking, "turns out that the people actually appreciated the theater"?
Is that in the same spirit of AC C?

Comments

  • faulknerfaulkner Member
    21 karma

    bruh that PT must be from 2042 how would i know

  • canihazJDcanihazJD Alum Member Sage
    8460 karma

    The flaw is the assumption that what happens in a specific instance happens in general... part to whole... true of subset vs true of group.

    Argument says they profited in the last week so they must have been profitable overall. C says these students who showed up oppose cuts so all students must oppose them too.

Sign In or Register to comment.